Search found 36 matches


Isn't the question asking (2^x)(3^y)(5^z)?

The way you wrote it: f(n) = 2^x3^y5^z it looks like everything is two is being raised with all of the powers or is it just me?

by adamsmith2009

Tue Nov 22, 2011 10:53 am
Forum: Problem Solving
Topic: functions
Replies: 10
Views: 1925

I'm confused. Isn't the question asking how much the store spent on writing pads? So, they made an inital purchase of 250 writing pads, which had cost the store $0.75 each plus an additional purchase (the only other one). So they want the total of the two purchases. A) Gives the number of additional...

by adamsmith2009

Thu Nov 17, 2011 3:33 pm
Forum: Data Sufficiency
Topic: amount spent
Replies: 9
Views: 1599

I don't understand how it could be solved with either. Can someone explain. Thanks.

by adamsmith2009

Mon Dec 14, 2009 6:57 am
Forum: Data Sufficiency
Topic: DS Groups
Replies: 2
Views: 1206

Yes - I think it's A. A) p-q>n If both p and q is positive no matter what numbers you choose it will prove that n is less than both. For example, p = 3 and q = 5 so p-q=-2 which is greater than n so n would have to be less than -2 Likewise if p = 5 and q = 3 the difference is 2 so 2>n so n would hav...

by adamsmith2009

Thu Dec 10, 2009 3:48 pm
Forum: Data Sufficiency
Topic: GMATPrep Question
Replies: 6
Views: 1763

E sounds like there is an obligation for the meteorite to strike the human being while D doesn't make that mistake.

by adamsmith2009

Thu Dec 10, 2009 3:22 pm
Forum: Sentence Correction
Topic: OG 11 SC
Replies: 2
Views: 1386

What if you have two integers? The question stem said only even number of integers.

Let's say (-1,10) or (-5,5) then you would have a negative median, no?

by adamsmith2009

Wed Dec 09, 2009 3:15 pm
Forum: Data Sufficiency
Topic: Set S
Replies: 8
Views: 2013

A) Not sufficient. X could be 3 which would make Y= 34 or X could be 2 which would make Y= 20. B) Sufficient. x(x-1) = multiple of Y So if x is 3 y is a multiple of 6. Therefore the minimum number y could be would be 6. So x can't be a multiple of y. If x is 5 y is a multiple of 20. Therefore the mi...

by adamsmith2009

Wed Dec 09, 2009 8:33 am
Forum: Data Sufficiency
Topic: function
Replies: 4
Views: 1368

1) x(x-1) = 20

2) 2^9

Explanation:

1. 3^X – 3^(X-1)= 162 then x(x-1) =


Factor: 3^(x-1)*(3-1) = 162

3^(x-1)(2) = 162

Divid 2 by both sides:

3^(x-1) = 81

Solve for x - 3^4 = 81. So x would be 5 and 5*4=20

2. 2+2+22+23+24+25+26+27+28
Again simplify = 2^9

by adamsmith2009

Tue Dec 08, 2009 5:41 pm
Forum: Problem Solving
Topic: GMAT Prep Que
Replies: 3
Views: 1273

Ahh - I see. I had the undergraduate ratio and not the graduate. Thanks Stuart.

by adamsmith2009

Thu Dec 03, 2009 5:16 am
Forum: Data Sufficiency
Topic: Weighted Average
Replies: 7
Views: 1914

Just want to confirm that the answer is 66% based on the weighted average method. 1 and 2 are not sufficient so if we combine:

G-----------------Avg----------------U
40 20

U/G = 40/20 = 4/2 = 4/6 or 66%

by adamsmith2009

Wed Dec 02, 2009 1:56 pm
Forum: Data Sufficiency
Topic: Weighted Average
Replies: 7
Views: 1914

IMO [/spoiler]D[spoiler]

A - 'doers evil' doesn't make sense.
B, C & E - use 'being'

That leaves D which is the best choice among the 5.

by adamsmith2009

Tue Dec 01, 2009 1:32 pm
Forum: Sentence Correction
Topic: Parallelism question please help...
Replies: 6
Views: 1301

But why wouldn't 4 work as well. If m = 2 that could be divisible by 4 and same if m = 3

by adamsmith2009

Tue Nov 24, 2009 5:00 pm
Forum: Problem Solving
Topic: Divide evenly problem
Replies: 6
Views: 2444

B Prime factor of 88,000 = 2^6*5^3*11 Blue = 1 Green = 5 Purple = X (X = 6, 7, 8, 9 or 10) Red = 11 We already hava 5 in Green and an 11 in Red and Blue doesn't matter since it's 1. We know that X = 6, 7, 8, 9 or 10 so the only value x could be based on 2^6 is 8^2 so 2 Purple chips were selected.

by adamsmith2009

Tue Nov 24, 2009 4:56 pm
Forum: Problem Solving
Topic: Tricky problem
Replies: 3
Views: 1364

Another way to do these types of questions is the formula: 2*S1*S2/S1+S2 = Average speed where S1 is the speed of the first trip and S2 is the speed of the second trip. This only works when the distance is split equally between the trips and there are no stops. I find this way to be much quicker.

by adamsmith2009

Mon Nov 23, 2009 3:25 pm
Forum: Problem Solving
Topic: cant figure out this OG question!
Replies: 10
Views: 8161

Yes - my mistake - X = 2 K = 0

by adamsmith2009

Mon Nov 23, 2009 3:14 pm
Forum: Data Sufficiency
Topic: Explain please
Replies: 8
Views: 1722